Difference between revisions of "2000 AMC 12 Problems/Problem 12"

(Solution)
m (Solution)
Line 6: Line 6:
 
== Solution ==
 
== Solution ==
 
It is not hard to see that  
 
It is not hard to see that  
<cmath>(A+1)(M+1)(C+1)=AMC+AM+AC+MC+A+M+C+1</cmath>
+
<cmath>(A+1)(M+1)(C+1)=</cmath>
 +
<cmath>AMC+AM+AC+MC+A+M+C+1</cmath>
 
Since <math>A+M+C=12</math>, we can rewrite this as
 
Since <math>A+M+C=12</math>, we can rewrite this as
 
<cmath>AMC+AM+AC+MC+13</cmath>
 
<cmath>AMC+AM+AC+MC+13</cmath>

Revision as of 14:40, 16 May 2015

Problem

Let $A, M,$ and $C$ be nonnegative integers such that $A + M + C=12$. What is the maximum value of $A \cdot M \cdot C + A \cdot M + M \cdot C + A \cdot C$?

$\mathrm{(A) \ 62 } \qquad \mathrm{(B) \ 72 } \qquad \mathrm{(C) \ 92 } \qquad \mathrm{(D) \ 102 } \qquad \mathrm{(E) \ 112 }$

Solution

It is not hard to see that \[(A+1)(M+1)(C+1)=\] \[AMC+AM+AC+MC+A+M+C+1\] Since $A+M+C=12$, we can rewrite this as \[AMC+AM+AC+MC+13\] So we wish to maximize \[(A+1)(M+1)(C+1)-13\] Which is largest when all the factors are equal. Since $A+M+C=12$, we set $A=B=C=4$ Which gives us \[(4+1)(4+1)(4+1)-13=112\] so the answer is $\boxed{E}$.

See also

2000 AMC 12 (ProblemsAnswer KeyResources)
Preceded by
Problem 11
Followed by
Problem 13
1 2 3 4 5 6 7 8 9 10 11 12 13 14 15 16 17 18 19 20 21 22 23 24 25
All AMC 12 Problems and Solutions

The problems on this page are copyrighted by the Mathematical Association of America's American Mathematics Competitions. AMC logo.png